Récurrence logarithmique d’ordre 2

Bonjour
Voici un défi posé à un camarade que je pensais avoir résolu il y a plusieurs années mais en fait je bloque encore sur un point. Ça devrait se débloquer si je m'y remets un jour mais peut-être aurez-vous un autre angle d'attaque.

On considère $(u_{n})$ définie par ses deux premiers termes strictement positifs, et pour tout entier $n$ : $$u_{n+2} = \log(1+u_{n}) + \log(1+u_{n+1}).

$$ Que dire de la convergence de cette suite ?

Réponses

  • Bonjour,

    $2.512862417252339...$ ?

    Cordialement,

    Rescassol
  • Par récurrence tous les termes sont entre 1 et 3 à partir d’un certain rang
    Puis avec des inégalités sur limite sup et limite inf voir la suite converge

  • Il va sans dire que la valeur de la limite éventuelle n'est pas l'essentiel (:P)

    Mais effectivement il y a convergence vers la limite possible non nulle pour toutes valeurs initiales. Ou du moins j'y miserais ma chemise.

    Etanche : je veux bien plus de détails.
  • En notant $\alpha$ l'unique solution de $2\ln(1+x) - x = 0$ dans $\left]0;+\infty\right[$, tu peux commencer par montrer que $n \mapsto \max(\alpha,u_n,u_{n+1})$ et $n \mapsto \min(\alpha,u_n,u_{n+1})$ sont respectivement décroissante et croissante, puis qu'elles ont la même limite.
  • RLC : alors tu as craqué, ça y est ? :-D
  • Pas mal side. Je regarde en détail.

    J'étais parti sur une étude de $|u_{n} - \ell|$, où $\ell$ est la limite possible.
    En remplaçant par l'équation qui caractérise $\ell$, et en utilisant une majoration du premier ordre du logarithme, j'avais une relation formelle du type : $$
    v_{n+2} < \frac{v_{n+1} + v_{n}}{1 + \ell},

    $$ où $v_{n} = \big|\frac{u_{n} - \ell}{1+ \ell}\big|$ si je ne dis pas de bêtises.

    Le problème de cette relation qui permettrait de conclure est que ma majoration de la valeur absolue du logarithme dépend du signe de $u_{n} - \ell$ et que donc mon identité est fausse en général.
    En considérant les deux suites $m_{n}$ et $M_{n}$ comme étant respectivement le minimum et le maximum de deux termes successifs, j'ai pu arriver à quelque chose dans tous les cas sauf un : celui où $m_{n} < \ell < M_{n}$ une infinité de fois.
    Ce qui ressemble peut-être à ce qu'a suggéré le poisson sympathique, sans que je sois capable d'arriver à bout.

    Edit : oui j'ai craqué :(
  • La même suite a été posté sur un autre forum , la limite s’exprime avec la fonction Lambert
    https://artofproblemsolving.com/community/c7h1912718p13103641

  • Encore une fois exprimer la limite n'est pas vraiment le problème intéressant.

  • Pas mal. J'aime bien le premier post, j'avoue que j'avais étudié la différence entre deux termes successifs mais sans penser à utiliser le fait que $u$ était minorée (ce qu'on obtient en cherchant à se convaincre heuristiquement que 0 ne peut pas être une limite envisageable) pour utiliser un argument de complétude.
  • Etanche: Pas besoin d’aller sur AoPS pour avoir la limite, WolframAlpha s’en charge très bien ! (ou à la main, si on connaît la fonction $W$).
  • Pourquoi ce n'est pas intéressant d'avoir une expression de la limite ?

  • C'est pas que ce n'est pas intéressant, c'est que c'est loin d'être le problème véritablement posé par cette étude (et en disant "la limite c'est tant" on donne l'impression que la convergence est le problème secondaire).
  • Heu ... si la suite a une limite, le problème de la convergence est bien traité, non ?

    Question : L'équation x²-6x+5=0 a-t-elle des solutions ?
    Réponse : on voit deux solutions évidentes, 1 et 5

    A-t-on bien répondu ?

    Cordialement.
  • Un peu de contexte : comme j'en ai ras le bol de ne jamais savoir faire des exos d'analyse réelle, j'ai ouvert plusieurs fils pour demander des exercices. RLC m'a passé cet exercice-là en message privé, et l'exercice constistait uniquement à démontrer que la suite est convergente. On trouve très facilement que la limite, si elle existe, ne peut être que le point fixe "intéressant" de $f$, donc on lui donne un nom et on essaie de démontrer la convergence.

    Il m'avait proposé cet exercice comme "version modifiée" d'un exercice qu'il avait fait en prépa (remplacer $\ln(1+x)$ par $\sqrt{x}$ et c'est bon), comme je galérais sur la version avec les logarithmes, il m'a donné la version de départ avec les racines à la place. Bon, je n'y suis pas arrivé non plus, en tout cas pas encore. Entre temps, il s'était rendu compte qu'il ne savait plus résoudre la version avec les logarithmes lui-même, je lui avais demandé qu'on se garde ça pour plus tard pour que je finisse mon exo tranquillement dans mon coin, mais je n'avance pas dessus (du moins, pas sans un coup de pouce à chaque étape, donc ça perd de son intérêt). Du coup, il a partagé l'exercice ici.

    Je suis d'accord que déterminer une expression "simple" (avec la fonction de Lambert, apparemment) du point fixe de la fonction est un exercice intéressant en soi, cela dit.
  • Attention,

    démontrer qu'il y a un point fixe n'est pas démontrer qu'il y a une limite. mais tu remettais en cause l'intérêt de prouver que la limite est l.

    Cordialement.

  • HT a tout dit, comme quoi il faut refaire un exo avant de le proposer à quelqu'un.

    Je crois que tu n'as pas compris le problème gerard.

    Edit : je remets en cause le fait qu'on vienne dire "la limite est l" en ayant l'air de dire que "c'est évident c'est le point fixe" comme sur le fil d'etanche.
    Une fois l'exercice d'analyse fait on peut s'amuser à faire des manipulations algébriques avec la fonction W si ça nous amuse mais c'est loin d'être le gros poisson.
    Un prof de sup nous péterait la gueule pour avoir fait ça.
  • $u_0>0,...,u_{p-1}>0$ , $u_{n+p}=\ln(1+u_n)+....+\ln(1+u_{n+p-1})$ doit aussi converger
  • Bonjour side et merci pour ta contribution (tu)

    Juste un mot, je n'ai pas compris ta récurrence immédiate $\forall n>2,\ |u_{n+2}-u_{n+1}|\le r^{n-2}|u_2-u_0|$ pourquoi $|u_{5}-u_{4}|\le r|u_2-u_0|$ ?
    Est-ce que quelqu'un peut m'expliquer la méthode de Simeon en détails.
    Merci.
    Le 😄 Farceur



  • Justement side, comme j'ai dit dans mon précédent post je suis parvenu à conclure si la suite est au-dessus de la limite (ou si les conditions initiales le sont vu qu'elle y reste), ou en-dessous tout le temps.
    C'est le cas où elle encadre la limite qui me bloque.

  • C'est très bien vu comme argument !

    Pour ceux que ça intéresse je posterai plus tard les deux cas au-dessus ou en-dessous de $l$.
  • Je détaille un peu.

    En étudiant $\phi : x \mapsto 2\ln(1+x) -x$, on constate qu'il existe un unique $\alpha > 0$ tel que $\phi(\alpha)=0$. De plus, pour tout $x > 0,\ \phi(x)$ est de signe opposé à $x-\alpha$.

    Pour tout $n \in \N$, notons $b_n = \max(\alpha, u_n, u_{n+1})$. Cette suite est trivialement minorée par $\alpha$, donc $2\ln(1+b_n) \leqslant b_n$ d'après l'étude de $\phi$. Par monotonie de $\ln$, on obtient alors facilement :
    $$
    \forall n \in \N,\quad u_{n+2} \leqslant 2 \ln(1+b_n) \leqslant b_n
    \quad\text{et}\quad
    b_{n+1} \leqslant \max(\alpha, u_{n+1}, b_n) \leqslant b_n.
    $$
    La suite $b$, décroissante et minorée par $\alpha$, converge donc vers un réel $\beta$ tel que $\beta \geqslant \alpha$.

    En considérant $u_{n+2}$ et $u_{n+3}$, on dispose aussi des inégalités :
    $$
    \forall n \in \N,\quad \alpha \leqslant b_{n+2} = \max(\alpha,u_{n+2},u_{n+3}) \leqslant \max(\alpha,2\ln(1+b_n), 2\ln(1+b_{n+1}))
    $$
    d'où $\alpha \leqslant \beta \leqslant \max(\alpha,2\ln(1+\beta))$ par passage à la limite. Donc $\beta \leqslant 2\ln(1+\beta)$, ce qui implique $\beta \leqslant \alpha$ d'après l'étude de $\phi$.

    Finalement, on a donc $\max(\alpha,u_n,u_{n+1}) \to \alpha$ et de même $\min(\alpha,u_n,u_{n+1}) \to \alpha$ en renversant le sens des inégalités. On conclut alors par encadrement.

    Remarque. On peut alléger la deuxième partie à l'aide des limites supérieures/inférieures en procédant comme https://math.stackexchange.com/a/574567/51594
  • Salut,

    On commence par montrer que la suite est bornée : on commence par choisir $m, M>0$ tels que $m< u_0,u_1\leqslant M$ et on va voir quelles conditions imposer à $M$ pour assurer que la suite est bornée. Supposons donc, par récurrence que $u_k \in [m,M]$ pour $k \leqslant n+1$, alors par monotonie de la fonction en jeu, $u_{n+2} \in [2\log (1+m),2\log(1+M)]$ et finalement on voit que la condition est que $2\log (1+M)\leqslant M$ et $m\leqslant 2\log (1+m)$ ce qui est satisfait pour $M$ assez grand et $m$ assez petit.

    Maintenant il s'agit de montrer que l'ensemble des valeurs d'adhérence est réduit à un seul nombre : notons $\ell, L$ les plus petites et plus grandes valeurs d'adhérence. Soit $u_{\phi(n)} \to L$, et quitte à réextraire, $u_{\phi(n)-1} \to \ell_1$, une autre valeur d'adhérence et $u_{\phi(n)-2}\to \ell_2$ une autre valeur d'adhérence, alors par la relation de récurrence, $L=\log(1+\ell _1)+\log(1+\ell_2)\leqslant 2\log(1+L)$ donc $L$ est plus petit que l'unique point fixe strictement positif de $x \mapsto \log(1+x)$. De même avec $\ell$, on a $\ell\geqslant 2\log(1+\ell)$.
  • Merci Simeon pour les détails. C'est plus clair.
    J'ai reçu une élégante réponse sur mathoverflow
    Le 😄 Farceur


  • J'avoue que la technique de Mickaël, je n'y aurais pas pensé : supposer qu'elle est bornée, puis trouver des conditions sur les bornes pour effectivement trouver des bornes.

  • C'est exactement la preuve que j'avais en tête à l'origine, avant de réaliser qu'il fallait considérer la position de la suite par rapport à $l$. Cependant, même sans avoir d'avance la convergence, ce raisonnement peur servir à l'établir dans le cas où les deux termes initiaux sont plus grands que $l$ (sans avoir besoin du epsilon donc).
    Merci pour ce post alors !

  • Pour le coup non, mais étudier cette différence en utilisant l'égalité définissant $l$ est je trouve l'idée la plus naturelle, et en voyant la difficulté l'étude des suites $m_{n}$ et $M_{n}$ est venue naturellement sans me permettre de conclure, mais les furieux de l'analyse du forum ont assez vite snipé la suite. Bravo à vous.
    C'est un exercice que j'avais trouvé sur le site de MP de Denis Monasse, de difficulté ++ d'après son auteur mais malgré tout dans une feuille censée faire office de révisions sur les suites niveau sup, pour les curieux.

    Dans tous les cas on a eu plusieurs jolies preuves sur ce topic, par la compacité ou par la méthode de Simenon. Merci à vous tous !
  • Une autre manière de résoudre le problème

    https://mathoverflow.net/questions/363180/convergence-for-a-non-linear-second-order-difference-equation

    @gebrane je ne savais pas que la main redirige vers un autre forum de maths
  • Cet exercice est corrigé dans le nouveau volume de Francinou Gianella Nicolas volume 3 analyse qui
    vient de sortir chez Cassini
    c ‘est à la page 170
  • Étanche Stp peux-tu scanner cetre page du corrigé
    Le 😄 Farceur


  • désolé je n’ai pas de scanner j’ai vu l’exos en passant chez Joseph Gibert cet après-midi.
  • avec ton téléphone ?
    Le 😄 Farceur


  • C’est le dernier livre volume 3 analyse nouvelle série
    http://store.cassini.fr/enseignement-des-mathematiques/103-oraux-x-ens-mathematiques-nouvelle-serie-vol-3.html

    Je compte bien l’acheter cet été
Connectez-vous ou Inscrivez-vous pour répondre.